You are on page 1of 11

Math 202 - Assignment 7

Authors: Yusuf Goren, Miguel-Angel Manrique and Rory Laster

Proposition 1 ([DF], p342). Let R be a ring and let M be an R-module. A subset N of M is an


R-submodule of M if and only if
1. N is nonempty and
2. x + ry N for all r R and all x, y N .
Exercise 10.1.4. Let R be a ring with identity, let M be the R-module Rn with component-wise
addition and multiplication, and let I1 , I2 , . . . , In be left ideals of R for some n N. The following are
submodules of Rn :
a. N1 = {(i1 , i2 , . . . , in ) : ik Ik for all k {1, 2, . . . , n}} and
Pn
b. N2 = {(x1 , x2 , . . . , xn ) : k=1 xk = 0}.
Proof. To prove (a), it suffices to show, by Proposition 1, that N1 is nonempty and x + ry N1 for
all r R and all x, y N1 . For the first condition, (0, 0, . . . , 0) N1 since Ik is a subgroup of R
containing the additive identity 0 for all k {1, 2, . . . , n}. That is, N1 is nonempty.
For the second condition, let x = (ik )kZ+ , y = (yk )kZ+ N1 and let r R. Then, by definition
of addition and scalar multiplication,
x + ry

(ik )kZ+ + r(yk )kZ+

(ik )kZ+ + (ryk )kZ+

(ik + ryk )kZ+

N1

since ik + ryk Ik for all k {1, 2, . . . , n} by the left ideal axioms. This gives
Pnus (a).
To establish (b), we apply a similar method as that used in (a). Since k=1 0 = 0, the element
(0, 0, . . . , 0) N2 . Thus N2 is nonempty. Moreover if x = (ik )kZ+ , y = (yk )kZ+ N1 and r R,
then
x + ry = (ik )kZ+ + r(yk )kZ+ = (ik + ryk )kZ+ .
Therefore, because
n
X

xk + ryk =

k=1

n
X

xk + r

k=1

= 0 + r0

n
X

!
yk

since I is a ring

k=1

since x, y N2

= 0,
we have that x + ry N2 by definition.
Exercise 10.1.5. Let R be a ring with identity, let I be a left ideal of R and let M be a left R-module.
Define

IM :=
ai mi : ai I and mi M for all i .

finite

IM is an R-submodule of M .

Proof. It suffices to show, by Proposition 1, that IM is nonempty and x + ry IM for all r R and
all x, y IM . For the former condition, observe that 0R I since I is an additive subgroup of R and
0M M because M is a group. Hence the finite sum 0R 0M = 0M satisfies the membership condition
of IM . Therefore IM is nonempty.
Pn
Pm
For the latter condition, let r R and let x = i=1 ai mi , y = i=1 a0i m0i IM such that n, m N,
ai , a0i I and mi , m0i M . Then
!
n
m
X
X
0 0
x + ry =
ai mi + r
ai mi
i=1

=
=

i=1

n
X

m
X
ai mi +
(ra0i )m0i

i=1
n
X

i=1
m
X

ai mi +

i=1

since M is a left R-module

a00i m0i

i=1

for a00i = ra0i I. That is, x + ry is a finite sum of elements of the form am such that a I and
m M . Thus x + ry IM .
Exercise 10.1.6. Let R be a ring with identity and let M be a left R-module. For any nonempty
collection {Ni }iI of R-submodules of M , the intersection
\
N=
Ni
iI

is an R-submodule of M .
Proof. Observe that N is a subset of M since, for all n N , n is an element of some Ni with i I.
Hence n M . So it suffices to show, by Proposition 1, that N is nonempty and x + ry N for all
r R and all x, y N . For the first property, 0M Ni for all i I because each Ni is an additive
subgroup of M . Therefore 0M N = iI Ni and, so, N is nonempty.
For the second property, let r R and let x, y N . Then x and y are elements of Ni for
all i I by definition. Thus, by the submodule axioms, x + ry Ni for each i I. That is,
x + ry N = iI Ni .
Exercise 10.1.7. Let R be a ring with identity and let M be a left R-module. If N1 N2 . . . is an
ascending chain of R-submodules of M , then
N=

Ni

i=1

is an R-submodule of M as well.
Proof. Suppose that N1 N2 . . . is an ascending chain of R-submodules of M . To prove that N
is also an R-submodule of M , it suffices to show, by Proposition 1, that N is nonempty and that
x + ry N for all r R and all x, y N .
Since 0 N1 , 0 is an element of the union N . Hence N is nonempty. For the remaining property,
let r R and let x, y N . Because x and y are elements of N , each must be an element of
a submodule. That is, x Nj and y Nk for some j, k N. By the ascending chain hypothesis,
Nmin(j,k) Nmax(j,k) . Therefore both x and y are members of Nmax(j,k) . Moreover, by the submodule
axioms, x + ry Nmax(j,k) . Hence, since Nmax(j,k) N , we have that x + ry N .
Definition. Let R be a ring and let M be a left R-module. A torsion element is an element m M
such that rm = 0 for some nonzero r R.

Definition. Let R be an integral domain and let M be a left R-module. The set
T or(M ) = {m M : m is a torsion element}
is the torsion submodule of M .
Exercise 10.1.8. Let R be a ring with identity and let M be a left R-module.
a. If R is an integral domain, then T or(M ) is an R-submodule of M ,
b. there exists a ring R with identity and a left R-module M such that T or(M ) is not a submodule of
M and
c. if R has zero divisors, then every nonzero left R-module contains nonzero torsion elements.
Proof. To prove (a), we suppose that R is an integral domain. It suffices to show, by Proposition 1,
that T or(M ) is nonempty and x + ry T or(M ) for all x, y T or(M ) and all r R.
For the former condition, 0 T or(M ) since 1 0 = 0. Hence T or(M ) is nonempty. For the final
condition, let x, y T or(M ) and let r R. As torsion elements, there exist nonzero s, t R such that
s x = 0 and t y = 0. Thus
(st) (x + ry) = (st) x + [(st)r] y

by the R-module axioms

= (ts) x + [(sr)t] y

by the commutativity of R

= t (s x) + (sr) (t y)

by the R-module axioms

= t 0 + (sr) 0

since s x = 0 and t y = 0

= 0.
Because R is an integral domain and s, t are nonzero, the product st is nonzero. Therefore, we have
shown that (st) (x + ry) = 0 for a nonzero st R. That is, x + ry T or(M ) and (a) is immediate.
To see that (b) holds, consider the ring R = M = Z/6Z and the elements 2, 3 R. R is a left
R-module with respect to addition and left ring multiplication. Moreover, since
23=6=0
and
3 2 = 6 = 0,
we find that 2 and 3 are elements of T or(M ). However, since 2 + 3 = 5 6 T or(M ), M is not closed
under addition. That is, T or(M ) is not a subgroup of M and, hence, it is not a submodule of M
either. Thus there exists a ring R and R-module M with the desired properties.
For (c), suppose that R contains the zero divisors s and r such that sr = 0. Then, for any
nonzero left R-module M with nonzero element m, either r m = 0 or r m 6= 0. In the first case of
r m = 0, m T or(M ) since r is nonzero by hypothesis. In the second case of r m 6= 0, we find that
r m T or(M ) because
s (r m) = (sr) m
=0m

by the R-module axioms


by hypothesis

= 0.
In either case, there exists a nonzero element contained in T or(M ). This is the desired result.
Definition. Let R be a ring with identity and let M be a left R-module. The annihilator of a submodule
N of M is the set
Ann(N ) = {r R : r n = 0 for all n N }.

Exercise 10.1.9. Let R be a ring with identity and let M be a left R-module. For any R-submodule
N of M , the annihilator of N in R is a two-sided ideal of R.
Proof. Suppose that N is an R-submodule of M , It suffices to show that Ann(N ) is nonempty and
that x + rys Ann(N ) for all x, y Ann(N ) and all r, s R. For the former condition, consider the
element 0R . Since
0R n = 0N
for any n N , we see that 0R Ann(N ).
For the remaining condition, we let x, y Ann(N ) and let r, s R. For any n N , we have that
(x + rys) n = x n + r (y (s n))
=0+r0

by the R-module axioms


since x, y Ann(N ) and sn N

= 0.
Hence x + rys Ann(N ).
Definition. Let R be a ring with identity and let M be a left R-module. The annihilator of an ideal
I of R is the set
Ann(I) = {m M : i m = 0 for all i I}.
Exercise 10.1.10. Let R be a ring with identity and let M be a left R-module. For any ideal I of R,
Ann(I) is an R-submodule of M .
Proof. Suppose that I is an ideal of R. To prove the desired result, it suffices to show, by Proposition
1, that Ann(I) is nonempty and x + ry Ann(I) for all x, y Ann(I) and all r R.
To see that Ann(I) is nonempty, observe that
i 0N = 0N
for all i I. Thus 0N Ann(I).
For the remaining condition, let x, y Ann(I) and let r R. If i I, then ir I by the ideal
axioms. Hence
i (x + ry) = i x + (ir) y
=0+0

by the R-submodule axioms


since x, y Ann(I) and ir I

= 0.
Therefore x + ry Ann(I).
Exercise 10.1.11. Let M be the Z-module Z/24Z Z/15Z Z/50Z.
a. Ann(M ) = 600Z and
b. Ann(2Z)
= GH such that G = h(12 + 24Z, 0 + 15Z, 0 + 50Z)i and H = h(0 + 24Z, 0 + 15Z, 25 + 50Z)i.
Proof. For (a), we appeal to the definition of set equality. To see the first inclusion, let a Ann(M ).
As it annihilates all elements of M , a must annihilate (1 + 24Z, 1 + 15Z, 1 + 50Z) M . That is,
0 = a (1 + 24Z, 1 + 15Z, 1 + 50Z) = (a + 24Z, a + 15Z, a + 50Z).
Since 0 = (0 + 24Z, 0 + 15Z, 0 + 50Z), the previous equation implies that

a + 24Z = 0 + 24Z,
a + 15Z = 0 + 15Z and

a + 50Z = 0 + 50Z.

Hence a 24Z 15Z 50Z = 600Z.


For the opposite inclusion, let a 600Z. Because 24, 15 and 50 are divisors of 600, observe that ab is
an element of each of 24Z, 15Z and 50Z for any b Z. Therefore, for all (x+24Z, y+15Z, z+50Z) M ,
a (x + 24Z, y + 15Z, z + 50Z)

(ax + 24Z, ay + 15Z, az + 50Z)

(0 + 24Z, 0 + 15Z, 0 + 50Z)

0.

Thus a annihilates all elements of M and, consequently, a Ann(M ) and we have now shown that
Ann(M ) = 600Z. This is (a).
For (b), we proceed similarly to the argument given in (a). To demonstrate the first inclusion, let
a = (x + 24Z, y + 15Z, z + 50Z) Ann(2Z). Then a annihilates all elements of 2Z and, in particular,
a annihilates 2. Thus
0 = 2 a = 2 (x + 24Z, y + 15Z, z + 50Z) = (2x + 24Z, 2y + 15Z, 2z + 50Z).
That is, we have the system of linear congruences

2x 0 (mod 24),
2y 0 (mod 15) and

2z 0 (mod 50).
By elementary number theoretic results, this system is equivalent to

x 0 (mod 12),
y 0 (mod 15) and

z 0 (mod 25).
Therefore x = 12x0 , y = 15y 0 and z = 25z 0 for some x0 , y 0 , z 0 Z. Moreover,
(x + 24Z, y + 15Z, z + 50Z)

(12x0 + 24Z, 15y 0 + 15Z, 25z 0 + 50Z)

(12x0 + 24Z, 0 + 15Z, 25z 0 + 50Z)

(12x0 + 24Z, 0 + 15Z, 0 + 50Z) + (0 + 24Z, 0 + 15Z, 25z 0 + 50Z)

G + H.

Hence Ann(2Z) G + H.
For the opposite inclusion, let a G + H. Then
a

n (12 + 24Z, 0 + 15Z, 0 + 50Z) + m (0 + 24Z, 0 + 15Z, 25 + 50Z)

(12n + 24Z, 0 + 15Z, 0 + 50Z) + (0 + 24Z, 0 + 15Z, 25m + 50Z)

for some n, m Z. For any element i in the ideal 2Z, it follows that i = 2j for some j Z. Thus
ia

(2j) [(12n + 24Z, 0 + 15Z, 0 + 50Z) + (0 + 24Z, 0 + 15Z, 25m + 50Z)]

(2j) (12n + 24Z, 0 + 15Z, 0 + 50Z) + (2j) (0 + 24Z, 0 + 15Z, 25m + 50Z)]

(24jn + 24Z, 0 + 15Z, 0 + 50Z) + (0 + 24Z, 0 + 15Z, 50jm + 50Z)]

(0 + 24Z, 0 + 15Z, 0 + 50Z) + (0 + 24Z, 0 + 15Z, 0 + 50Z)]

0.

Hence a annihilates all elements of 2Z and, therefore, a Ann(2Z). This establishes the opposite
inclusion of G + H Ann(2Z) and, so, Ann(2Z) = G + H. Furthermore, since G H = {0}, it follows
that G + H
= G H and, so, Ann(2Z)
= G H.

Exercise 10.1.14. Let R be a ring with identity and let M be a left R-module. For any element z of
the center of R,
zM = {zm : m M }
is an R-submodule of M .
Proof. Let z be an element of the center of R. It suffices to show, by Proposition 1, that zM is
nonempty and x + ry zM for all x, y zM and all r R. Since 0M M ,
z 0M = 0M zM.
That is, zM is nonempty.
For the remaining submodule criterion, let x = zm, y = zm0 zM and let r R. Then
x + ry = z m + r (z m0 )
= z m + (rz) m0

by the R-submodule axioms

= z m + (zr) m

since z is in the center of R

= z (m + r m )

by the R-submodule axioms.

Therefore, because m + r m0 M , we have that x + ry zM .


Corollary. Let R = M2 (F ) for a field F and let


1 0
e=
M2 (F ).
0 0
Then e is not in the center of R.
Proof. Consider R as a left R-module in the natural way. The element e = e 1R is a member of R
and, also,


1 0
r=
R.
1 0
Since
re =


1
1

 
0
1

0
0

 
0
1
=
0
1


0
=r
0

and r 6 eR, we find that eR is not closed under scalar multiplication. Therefore eR is not a submodule
of R. This implies, by the contrapositive to Exercise 10.1.14, that e is not in the center of R.
Exercise 10.1.16. Let V = F n for a field F and let T : V V be the shift operator defined by
T (v1 , v2 , . . . , vn ) = (v2 , . . . , vn , 0).
If we View V as an F [x]-module with x acting by the operator T and if U is an F [x]-submodule of V ,
then U = Uk for some Uk = Span({ei : 1 i k}) such that k {0, 1, . . . , n}.
Proof. The abelian group V = F n is a left F [x]-module when endowed with a scalar multiplication
: F [x] V V defined by
!
m
m
X
X
i
ai x v =
ai T i (v)
i=0

Pm

i=0

for all i=0 ai x F [x] and all v V .


With this definition in mind, suppose that U is an F [x]-submodule of V . There exists a maximal
element in the set of integers
S = {m Z : there exists an element u U such that u has nonzero mth coordinate}

because there are only n coordinates in any element of V . Let k be this maximal integer of S. That
is, for all u U and all l > k, the lth coordinate of u is equal to 0.
We claim that U = Uk for this maximal k. To see this, observe that U is closed under scalar
multiplication by F [x] since it is a submodule. Hence, if f is a nonzero element in the kth coordinate
of an element u = (u1 , . . . , uk1 , f, 0, . . . , 0) U , then
(f 1 xk1 ) u = f 1 T k1 (u)
= f 1 (f, 0, . . . , 0)
=

(1, 0, . . . , 0)

= e1 .
Thus e1 U .
Furthermore, since U is also closed under addition,
(f 1 xk2 ) u (f 1 uk1 ) e1

= f 1 T k2 (u) (f 1 uk1 , 0, . . . , 0)
= f 1 (uk1 , f, 0, . . . , 0) (f 1 uk1 , 0, . . . , 0)
=

(0, 1, 0, . . . , 0)

= e2 .
So e2 U .
Continuing in this way, we find that ei U for all i {1, 2, . . . , k}. Therefore Span({ei : 1 i
k}) = Uk is contained in U by the F [x]-submodule properties. That is, Uk U . Moreover, since k
is the maximal coordinate for which there is an element with nonzero entry, it follows that U Uk .
Hence U = Uk , as desired.
Exercise 10.1.18. Let F = R, let V = R2 and let T EndR (V ) be the map such that


0 1
T (v) =
v
1 0
for all v V . Then V and 0 are the only F [x]-submodules of V with respect to T .
Proof. Let U be an F [x]-submodule of V . Since F embeds into F [x], we may view V as an F -vector
space and U as an F -subspace of V . Either U = 0, U = V or the dimension of U as an F -subspace is
equal to 1. If dimR (U ) = 1, then there exists a nonzero element u U such that U = spanR (u). For
U to be an F [x]-submodule of V , we must have that T (U ) U by the discussion in 10.1 of [DF]. In
particular, T (u) = u for some R. Therefore, by definition, u is an eignevector of T with a real
eigenvalue. However, the eigenvalues of T are i and, so, T has no real eigenvalues. This contradiction
implies that such a U cannot occur. That is, U = {0} or U = V .
Exercise 10.1.19. Let F = R, let V = R2 and let T EndR (V ) be the map defined by T (a, b) = (0, b).
Then the set
S = {W V : W is an F [x]-submodule of V }
satisfies S = {0, Wx , Wy , V } such that Wx = {(a, 0) V : a R} and Wy = {(0, b) V : b R}.
Proof. The set of F [x]-submodules is equal to the set
S = {W V : W is a subspace of V and W is T -stable}
by an argument given in 10.1 of [DF].
For the subspace Wx of V ,
T (a, 0) = (0, 0) Wx
for all (a, 0) Wx . Hence Wx is T -stable and, moreover, Wx S.

Similarly, for the subspace Wy ,


T (0, b) = (0, b) Wy
for all (0, b) Wy and, consequently, Wy S. It follows that, since 0 and V are trivially contained in
S, {0, Wx , Wy , V } S.
To demonstrate the opposite inclusion, we let W S. There are two cases of W : either W contains
an element (a, b) such that both a and b are nonzero or it does not. In the first case of the existence
of such an element (a, b), we find that, by the T -stability of W ,
T (u) = T (a, b) = (0, b) W.
But (a, b) and (0, b) are R-linearly independent since


a b
det
= ab
0 b
and a and b are nonzero by hypothesis. Since it is a subspace of the two-dimensional R-vector space
V containing two linearly independent elements, W must equal V .
In the second case where no elements exist in W of the form (a, b) such that both a and b are
nonzero, it follows that W is either equal to 0, Wx or Wy . Thus W {0, Wx , Wy , V }, as desired.

Lemma 1. Let A be a Z-module, let a A and let n be a positive integer. The map a : Z/nZ A
defined by a (k) = ka is a well-defined Z-module homomorphism if and only if na = 0.
Proof. Suppose that a is a well-defined Z-module homomorphism. In the Z-module Z/nZ, n = 0.
Hence, because a is well-defined,
na = a (n) = a (0) = 0a = 0.
That is, na = 0.
For the converse statement, suppose that na = 0 and consider the map : Z A defined by
(k) = ka for all k Z. This map is a group homomorphism since
(k1 + k2 ) = (k1 + k2 )a
= k1 a + k2 a

by the Z-module axioms

= (k1 ) + (k2 )
for all k1 , k2 Z. Furthermore, because
(nk) = (kn)
= (kn)a
= k(na)

by the Z-module axioms

= k0

since na = 0 by hypothesis

=0
for all nk nZ, we find that the subgroup nZ is contained in the kernel of . Therefore descends
to the quotient Z/nZ. That is, there exists a unique group homomorphism a : Z/nZ A such that
a (k) = (k) = ka. Moreover, since
a (k1 k2 ) = a (k1 k2 )
= (k1 k2 )a
= k1 (k2 a)

by the Z-module axioms

= k1 a (k2 )
for all k1 Z and all k2 Z/nZ, it follows that a is a Z-module homomorphism.

Exercise 10.2.4. Let A be a Z-module and let n be a positive integer. Then HomZ (Z/nZ, A)
= An
where An = {a A : an = 0}.
Proof. Let HomZ (Z/nZ, A) and let a = (1). For any k Z+ , that is a Z-module homomorphism
implies
!
!
k
k
k
k
X
X
X
X
(k) =
1 =
(1) =
a=
1 a = ka.
i=1

i=1

i=1

i=1

Thus = a , with a defined as in Lemma 1.


By Lemma 1, = a is a Z-module homomorphism if and only if na = 0. Therefore the function : HomZ (Z/nZ, A) An defined by (a ) = a is a bijection. Furthermore is a Z-module
homomorphism since
(a + a0 )

= (a+a0 )
= a + a0
= (a ) + (a0 )

and
(ka )

(ka )

ka

k(a )

for all k Z and all a , a0 HomZ (Z/nZ, A). That is, is an isomorphism and we have that
HomZ (Z/nZ, A)
= An .
Exercise 10.2.5. Let a Z/21Z and let a : Z/30Z Z/21Z denote the map defined by a (k) = ka.
Then
HomZ (Z/30Z, Z/21Z) = {0 , 7 , 14 }.
Proof. By Exercise 10.2.4, the map : An HomZ (Z/30Z, Z/21Z) defined by (a) = a is an
isomorphism for An = {a Z/nZ : 30a = 0}. So it suffices to show that An = {0, 7, 14}.
For the set relation An {0, 7, 14}, we let a An . Then
30a = 30a = 0 = 0

(1)

since a annihilates 30 by the membership condition of An . Thus, because 30a and 0 are elements of
Z/21Z, (1) implies that
30a 0 (mod 21).
So, by definition, 21 divides 30a and there exists b Z such that 21b = 30a. Dividing this equation
by 3 yields that 7b = 10a and, because 7 is coprime to 10, we find that 7 must divide a. That is,
a {0, 7, 14}.
For the opposite inclusion, we let a {0, 7, 14}. Then 7 divides a and it is clear that, for some
b Z,
30a = 307b = 30 7b = 21 10b = 0 = 0.
Hence a An . Therefore An = {0, 7, 14}, as required.
Exercise 10.2.6. For all positive integers m and n,
HomZ (Z/nZ, Z/mZ)
= Z/(n, m)Z.

Proof. Let m and n be positive integers. By Exercise 10.2.4,


HomZ (Z/nZ, Z/mZ)
= An
Z/(n, m)Z.
where An = {a Z/mZ : na = 0}. So it suffices to show that An =
To see that this is the case, consider an arbitrary element a An . As an element of An , a annihilates
n and, therefore,
na = na = 0 = 0.
That is, na = 0 in the group Z/mZ. Thus
na 0

(mod m)

to imply that there exists an integer k such that km = na. Furthermore, we find by dividing by the
greatest common divisor (n, m) that
n
m
=
.
k
(n, m)
(n, m)
So

m
(m,n)

n
(n,m) a. It follows,
n
hence, (n,m)
must divide

divides the product

n
(n,m)

are coprime and,


Therefore, we have shown that


An

m
by elementary number theoretic results, that (n,m)
and
n
o
km
m
a. Since a is divisible by (n,m) , a (n,m) : 1 k (n, m) 1 .


km
: 1 k (n, m) 1 .
(n, m)

The opposite inclusion follows from the observation that each of the implications
n in the derivation of
o
km
the previous set relation are, in fact, equivalences. Or, more pedantically, let a (n,m)
: 1 k (n, m) 1 .
Then

m
(n,m)

divides a and, so, a =

k0 m
(n,m)

for an integer k 0 . Hence

(n, m)a = (n, m)

k0 m
= k0 m 0
(n, m)

(mod m).

Consequently (n, m)a = 0 and we find that a An . This gives us that




km
: 1 k (n, m) 1
(n, m)


An .

Each set inclusion holds and we conclude that




km
An =
: 1 k (n, m) 1 .
(n, m)
Moreover An is isomorphic to Z/(n, m)Z by the map : An Z/(n, m)Z defined by
That is, An
= Z/(n, m)Z and the proof is complete.

km
(n,m)

= k.

Exercise 10.2.9. Let R be a commutative ring. Prove that HomR (R, M ) and M are isomorphic as
left R-modules.
Proof. Throughout this problem let a, r, r0 R and m, m0 M . We follow the hint. Let ,
HomR (R, M ). Suppose (1) = (1) = m. We must have
(a 1) = a(1) = am

and
(a 1) = a(1) = am.

Since a was arbitrary, = . This shows that any HomR (R, M ) is equal to m for some m.
Define f : HomR (R, M ) M ; m 7 m. The map is obviously well-defined. To see that f is an
R-module homomorphism, first observe
(m + rm0 )(1) = m (1) + rm0 (1) = m + rm0 = m+rm0 (1).
Hence
f (m + rm0 ) = f (m+rm0 ) = m + rm0 = f (m ) + rf (m0 ).
To see that f is injective, suppose f (m ) = 0. This implies m = 0. It follows that m (r) = 0 for all
r. This shows m = 0 HomR (R, M ), i.e., the kernel is trivial. Finally, f is surjective as well. To
this end we will first show that the map m : R M given by 1 7 m and r 7 rm is an R-module
homomorphism:
m (r + ar0 ) = (r + ar0 )m = rm + ar0 m = m (r) + am (r0 ).
Therefore given m M , there exists m HomR (R, M ) such that f (m ) = m, and the proof is
complete.
Exercise 10.2.11. Let A1 , . . . , An be R-modules and let Bi be a submodule of Ai for each i = 1, . . . , n.
Prove that
(A1 An )/(B1 Bn )
= (A1 /B1 ) (An /Bn ).
Proof. Define an R-module homomorphism : A1 An (A1 /B1 ) (An /Bn ) by
(a1 , . . . , an ) = (a1 , . . . , an ), where we let ai denote the image of ai Ai in the quotient Ai /Bi .
It is easy to check that this is indeed an R-module homomorphism. Furthermore, it is clearly surjective. The kernel consists of exactly the tuples (a1 , . . . , an ) such that ai = 0 for all i, i.e. such that
ai Bi . Thus the kernel is B1 Bn , and hence we obtain the desired result from the first
isomorphism theorem.
Exercise 10.2.13. Let I be a nilpotent ideal in a commutative ring R, let M and N be R-modules and
let : M N be an R-module homomorphism. Show that if the induced map : M/IM N/IN is
surjective, then is surjective.
Proof. Since is surjective, N/IN = (M/IM ) = ((M ) + IN )/IN . By the lattice isomorphism
theorem for modules, N = (M ) + IN .
We will show N = (M ) + I t N for t 1 by induction. The base case is already shown. Suppose
the equation holds for some t 1. Then
N = (M ) + I t N = (M ) + I t ((M ) + IN ) = (M ) + I t (M ) + I t+1 N
= (M ) + I t N.
Since I k = 0 for appropriate k, the conclusion follows.

References
[DF]

Dummit, David and Foote, Richard. Abstract Algebra, 3rd edition.

You might also like